1

This is my first post. I'm not a mathematician, just an electronics engineer who loves mathematics. In one of my projects, I arrived at the following function:

$$V\left(\varphi\right)=\frac{A\sqrt{\pi-\varphi+\sin{\varphi\cos{\varphi}}}}{\sqrt{2\pi}}$$

The project requires $V\left(\varphi\right)$ to be inverted, to obtain angle $\varphi$ (unknown), from a voltage $V$ (known). $V\left(\varphi\right)$ is continuous and strictly monotonic (descending), so an inverse mapping should exist. I tried to invert it symbolically, but couldn't arrive at a closed-form solution for $\varphi\left(V\right)$. By "closed-form solution" I mean a formula that takes me directly (algebraically) from $V$ to $\varphi$. I ended up using MATLAB to compute it numerically, and the project was successfully completed. That was 11 years ago (in 2009).

Recently, that inversion problem resurfaced. Out of pure curiosity, I asked my cousin (a mathematician) to attempt to symbolically invert the above function, but he also couldn't do it, and couldn't even give me an answer as to the existence of such solution. So, my questions are the following:

  1. Does a closed-form expression for $\varphi\left(V\right)$ exist?
  2. If the answer to (1) is YES, can someone provide that function, or point me to a method for deriving it?
  3. If the answer to (1) is NO, what is the formal reason for it? Is there a way to show/prove that such solution does not exist?

This is not a homework question, and the associated practical problem has already been solved numerically. This post was made out of pure curiosity about the invertibility of functions of the form of $V\left(\varphi\right)$. Many thanks to all for your replies.


Adding some graphics, in order to better illustrate the problem.

The following figure shows how the function we're looking for, $\varphi\left(V\right)$, looks like. Notice that there appears to be no symmetry in this function. The values of $\varphi$ lie in the interval $[0,\pi]$, while the values of $V\left(\varphi\right)$ lie in the interval $[0,\frac {A} {\sqrt2}]$.

enter image description here

Based on the comment by @user2661923, the problem can be reduced to the inversion of function $k=x-\sin(x)$. In this case, the values of both $x$ and $k$ lie in the interval $[0,2\pi]$. The following figure shows a plot of the inverse of $k=x-\sin(x)$, together with a plot of the function itself (dashed line). Plotting them both on the same graph is useful, since they both have the same range for their independent and dependent variables. It can be observed that now there is a clear symmetry of this function at the point $(\pi,\pi)$, thanks to the removal of the square root term. This means that we only need to deal with the interval $[0,\pi]$, and use symmetry on that result to obtain the other half ($[\pi,2\pi]$).

enter image description here

  • 1
    Let $B ~=~$ $\displaystyle\left[\frac{\sqrt{2\pi} \times V(\varphi)}{A}\right]^2$. Let $C = B - \pi.$ Then $C = \sin \varphi \cos \varphi - \varphi.$ The problem can be reduced still further via $\alpha = 2\varphi ~\Rightarrow ~ 2C = \sin \alpha - 2\alpha.$ I am unaware of any function that permits a closed form evaluation of $x$ based on $2C = \sin x - 2x.$ – user2661923 Oct 05 '20 at 16:28
  • Re previous comment : typo, should be ...$2C = \sin \alpha - \alpha.$ ... $2C = \sin x - x.$ – user2661923 Oct 05 '20 at 17:23
  • Thanks for your reply, I appreciate it. I also could not find a way to invert a function of the form $\sin\alpha - \alpha$. I wonder if there is a formal proof that such functions do not have a closed-form inverse. Maybe it's something known, that I'm not aware of—I'm not a mathematician. Hopefully, someone can provide such a proof, or some helpful information about this issue. – Costas Vlachos Oct 06 '20 at 05:06
  • Done. Have a look at my second answer. Please tell me how this works for you. Cheers :-) – Claude Leibovici Oct 08 '20 at 03:46
  • 1
    This is not about the particular function you are interested in but it shows you the tools that are used in proving the relevant impossibility results. If you get comfortable with the text linked to below, I'll post a link to one more about the inverse function to $e^{x}+e^{ax}$ (which is not too far from your case but results in a first order differential relation for the (derivative of) the inverse function; yours looks like a second order one, which is generally harder to handle). So, start with https://ksda.ccny.cuny.edu/PostedPapers/liouv06.pdf – fedja Oct 09 '20 at 00:37
  • Also posted to MO, https://mathoverflow.net/questions/373513/invertibility-of-specific-function – Gerry Myerson Oct 09 '20 at 06:49

5 Answers5

2

Using @user2661923's comments, we end with $$k=x-\sin(x)$$

The first thing we could do is a Taylor expansion around $x=0$ ( to get $$k=\frac{x^3}{6}-\frac{x^5}{120}+\frac{x^7}{5040}-\frac{x^9}{362880}+\frac{x^{11}}{39 916800}-\frac{x^{13}}{6227020800}+O\left(x^{15}\right)$$ which is very accurate.

Then a series reversion leads to
$$x=t+\frac{t^3}{60}+\frac{t^5}{1400}+\frac{t^7}{25200}+\frac{43 t^9}{17248000}+O\left(t^{11}\right)$$ where $t=\sqrt[3]{6k}$.

Edit

Doing the same around $x=\frac \pi 2$ , we should get $$x=\frac{\pi }{2}+t-\frac{t^2}{2}+\frac{t^3}{2}-\frac{7 t^4}{12}+\frac{3 t^5}{4}-\frac{46 t^6}{45}+\frac{29 t^7}{20}-\frac{21341 t^8}{10080}+O\left(t^{9}\right)$$ where $t=k+1-\frac \pi 2$.

Doing the same around $x=\pi $ , we should get $$x=\pi +t+\frac{t^3}{12}+\frac{t^5}{60}+\frac{43 t^7}{10080}+\frac{223 t^9}{181440}+\frac{60623 t^{11}}{159667200}+\frac{764783 t^{13}}{6227020800}+O\left(t^{15}\right)$$ where $t=\frac{k-\pi}2$.

  • Thanks Claude, this is a clever approach to the inversion, although it doesn't really answer my questions. Still, your answer has practical significance. I've tested it, and the results show a good match to the true inverse for angle values close to $\varphi = 0$ (as expected), but it completely breaks down when $\varphi > \pi/2$, which is unacceptable from an engineering point of view. You can clearly see the deviation in this graph. Things should improve if you do the Taylor expansion around $\varphi = \pi/2$ (centre). Note that $0\leq\varphi\leq\pi$. – Costas Vlachos Oct 06 '20 at 19:42
  • @DrCeeVee. Answer edited. Please, let me know how it works. Cheers :-) – Claude Leibovici Oct 07 '20 at 04:20
  • Thanks again Claude. You've now done the Taylor expansion around $\varphi=\pi/4$, instead of $\varphi=\pi/2$. This is because $x=2\varphi$. To do it properly, you need to do it around $x=\pi$. Now there is a good match around $\varphi=\pi/4$ (as expected), but it deviates elsewhere, as shown in this graph. If possible, please redo the Taylor expansion around $x=\pi$ (which is around $\varphi=\pi/2$) and, perhaps, include a few more terms, so that it doesn't deviate too much. Somehow I feel that the Taylor expansion method won't help us much though... – Costas Vlachos Oct 07 '20 at 12:59
  • Just tested your last result (Taylor expansion around $x=\pi$), and it looks great around $x=\pi$ (as expected), while providing much better accuracy at the edges, when compared to the previous two cases. This is clearly illustrated in this graph. Not trying to be picky, but I'm afraid it's still unacceptable for practical engineering applications. The error is around 27 V (12%) at $\varphi=\pi$. I think this is a useful result, that gives us an idea of what to expect from the Taylor expansion approach. Many thanks again Claude, I really appreciate it. – Costas Vlachos Oct 07 '20 at 14:09
  • @DrCeeVee. I have something which is not bad. I shall finish tomorrow (dinner time here). Cheers – Claude Leibovici Oct 07 '20 at 16:07
  • Question: how did you compute the inversions of the Taylor expansions? Are you using a symbolic computation software like Mathematica? BTW, just saw your new method. Will check it out soon and let you know how it compares with the others, thanks. – Costas Vlachos Oct 09 '20 at 14:54
1

First of all, most algebraic expressions do not have nice closed-form inverses. Even polynomials of degree 5 or greater will generally not have one, except by use of really esoteric functions that are no easier to calculate than those numeric methods you've mentioned.

In this case, you would be fine except for that mixture of $\phi$ inside and outside of the trigonometric functions. As user2661923 points out, this part can be simplified to $\sin\alpha - \alpha$, but that has no nice inverse. Mind you, You can define a function to give that inverse, but this is (mostly) just linguistic shuffling, not actual solving.

Just say "Define $w(y)$ to be the angle $\alpha$ for which $\sin \alpha - \alpha = y$". Now you can solve your equation thusly:

$$\phi = \frac 12w\left(\frac {2\pi V^2}{A^2} - \pi\right)$$

Looks nicer, but from a standpoint of calculation, you are not really any closer. If you are just looking for a few values, a judicious use of Newton's method will find them faster.

However, if you have need to make this calculation repeatedly, then it makes sense to find enough terms of the Taylor series of $w$ to get the accuracy you need. The inverse of $w$, that is $\sin \alpha - \alpha$ has a well-known Taylor series, and there are techniques for inverting power series. Or you can use implicit differentiation to find derivatives of $w$ and build the Taylor series that way.

Paul Sinclair
  • 43,643
  • Many thanks Paul, for your detailed answer, and also that of @user2661923. This confirms my suspicion that a closed-form expression for $\varphi\left(V\right)$ does not exist. I just wonder if there is a formal reason why this is so, i.e., something like "functions of the form $\sin\alpha - \alpha$ do not have a closed-form inverse", and to see the proof of that statement (or a link to some work that proves it). I will wait for a little while in case someone provides this, and then I will mark your post as an accepted answer. – Costas Vlachos Oct 06 '20 at 04:49
1

I prefer to add a second answer.

If series reversions are not sufficiently good, the only practical posibility I can see is to express $\big[x-\sin(x)\big]$ as a rational function easy to inverse. So, the idea was to try to reduce the problem to a quadratic equation in some power of $x$.

From that, the idea of using $$f(x)=x-\sin(x) \sim \frac {x^n(a+b x^n)}{1+cx^n+d x^{2n}}=g_n(x)$$ Matching the function and first derivative values at $x=0,\frac \pi 2,\pi$,was computed $$\Phi_k=\int_0^\pi \Big[f(x)-g_n(x)\Big]^2\,dx$$ What was obtained is $$\Phi_1=2.33\times 10^{-4} \qquad \Phi_2=7.33\times 10^{-5} \qquad \Phi_3=1.25\times 10^{-6}$$ So, $k=3$ was retained (it leads to a maximum error of $0.0015$). Notice that this choice is also dictated by the fact that, plotted as function of $x$, $\big[x-\sin(x)\big]^{\frac 13}$ is very close to linearity.

Then, the problem is just a quadratic equation in $x^3$ $$ (b-d k)x^6+ (a-c k)x^3-k=0$$

$$x=\left(\frac{\sqrt{(a-c k)^2+4 k (b-d k)}-(a-c k)}{2 (b-d k)}\right)^{\frac 13}$$

The parameters are

$$a=\frac{819 \pi ^2-4704 \pi+6720}{2\pi^ 2(31 \pi^2 -72\pi-72) }\qquad \qquad b=-\frac{42 (\pi^2-56\pi+ 164)}{\pi ^5 (31 \pi^2 -72\pi-72)}$$ $$c=\frac{3 (395 \pi^2 -1472\pi+768)}{2 \pi ^3 (31 \pi^2 -72\pi-72)}\qquad \qquad d=-\frac{8 (32 \pi^2 -285\pi+576)}{\pi ^6 (31 \pi^2 -72\pi-72)}$$

Below are reported some results

$$\left( \begin{array}{ccc} k & \text{estimate} & \text{solution} \\ 0.0 & 0.00000 & 0.00000 \\ 0.1 & 0.85428 & 0.85375 \\ 0.2 & 1.08378 & 1.08369 \\ 0.3 & 1.24849 & 1.24852 \\ 0.4 & 1.38226 & 1.38228 \\ 0.5 & 1.49729 & 1.49730 \\ 0.6 & 1.59958 & 1.59959 \\ 0.7 & 1.69257 & 1.69259 \\ 0.8 & 1.77844 & 1.77851 \\ 0.9 & 1.85867 & 1.85881 \\ 1.0 & 1.93434 & 1.93456 \\ 1.1 & 2.00623 & 2.00655 \\ 1.2 & 2.07496 & 2.07538 \\ 1.3 & 2.14100 & 2.14151 \\ 1.4 & 2.20474 & 2.20534 \\ 1.5 & 2.26650 & 2.26717 \\ 1.6 & 2.32653 & 2.32726 \\ 1.7 & 2.38505 & 2.38584 \\ 1.8 & 2.44227 & 2.44308 \\ 1.9 & 2.49833 & 2.49915 \\ 2.0 & 2.55339 & 2.55420 \\ 2.1 & 2.60757 & 2.60834 \\ 2.2 & 2.66097 & 2.66169 \\ 2.3 & 2.71370 & 2.71436 \\ 2.4 & 2.76585 & 2.76643 \\ 2.5 & 2.81751 & 2.81799 \\ 2.6 & 2.86873 & 2.86912 \\ 2.7 & 2.91961 & 2.91989 \\ 2.8 & 2.97019 & 2.97038 \\ 2.9 & 3.02054 & 3.02065 \\ 3.0 & 3.07073 & 3.07077 \\ 3.1 & 3.12079 & 3.12080 \end{array} \right)$$

Using as a basis this model, a nonlinear regression gives the following results $(R^2 > 0.9999999)$ $$\begin{array}{clclclclc} \text{} & \text{Estimate} & \text{Standard Error} & \text{Confidence Interval} \\ a & 0.164158286954 & 0.000056596 & \{0.164045944,0.164270630\} \\ b & 0.033111490794 & 0.000405629 & \{0.032306324,0.033916658\} \\ c & 0.241175528209 & 0.002683157 & \{0.235849503,0.246501553\} \\ d & 0.003405195311 & 0.000043215 & \{0.003319414,0.003490977\} \\ \end{array}$$

Making them rational $$a=\frac{5061}{30830} \qquad b=\frac{403}{12171}\qquad c=\frac{1551}{6431}\qquad d=\frac{148}{43463}$$ which are quite close to the theoretical values given above (but they better distribute the errors, the maximum of them being $0.0002$).

  • Once more, many thanks Claude. I tried your last approach. The results were close to the true original, but worse than the last Taylor approximation. See this graph, which shows all 3 plots. Please note that $0\leq k \leq 2\pi$. In your table of numerical values, you test $k$ in the interval $[0, \pi]$, which is half the range of values that $k$ takes! If possible, please test $k$ in the interval $[0, 2\pi]$ and see what error you're getting, because the plot shows large deviations for $k > 4$ (which corresponds to $\varphi > 2$ in the plot). – Costas Vlachos Oct 08 '20 at 08:09
  • @DrCeeVee. May I say that I totally disagree : for $0 \leq x \leq \pi$, the fit is almost perfect (look at the statistics I added). For $ \pi \leq x \leq 2\pi$, you must take into account the symmetry. – Claude Leibovici Oct 08 '20 at 08:30
  • Maybe that is the source of the problem. I'm not taking any symmetry into account, I'm just evaluating your expression in the interval $0\leq k \leq 2\pi$. You're absolutely right, the match is perfect when $0\leq k \leq \pi$. So, as I understand it, what you're saying is that I should split the equation into two branches, one for $0\leq k \leq \pi$, and another for $\pi < k \leq 2\pi$, taking into account the symmetry of $\sin x$. Let me try this, and I will get back to you very soon. Thanks again for your work, I really appreciate your help, regardless of the outcome! – Costas Vlachos Oct 08 '20 at 08:57
  • Also, note that the independent variable is $k$, not $x$. So, for the solution to have any practical significance, we need to arrive at a function of the form $x=f(k)$, where $0\leq k \leq 2\pi$, that will match the true inverse, as shown in the blue plots in the graphs I posted. How would you achieve that, based on your last solution? I'm stressing that $k$ should go from $0$ to $2\pi$, not from $0$ to $\pi$. If you can, please provide me with that expression, taking the symmetry into account. If it's too much trouble, I will try to work it out myself. Once again, many thanks for your help. – Costas Vlachos Oct 08 '20 at 09:17
  • @DrCeeVee. I gave you $x=f(k)$ – Claude Leibovici Oct 08 '20 at 09:33
  • Just to add one clarification and one correction: in my comment above, when I'm saying "the independent variable is $k$, not $x$" I'm obviously referring to the solution (i.e., the inverse function). Also, in an earlier comment, I'm saying "the plot shows large deviations for $k > 4$ (which corresponds to $\varphi > 2$ in the graph)". This is incorrect. The correct value of $\varphi$ that corresponds to $k = 4$ is $\varphi \approx 1.79$. It still supports the point I was trying to make though. – Costas Vlachos Oct 08 '20 at 09:44
  • Yes, of course you gave me $x = f(k)$. The point is that your function breaks down when $k$ approaches $2\pi$. I think that my mistake was not taking care of the symmetry around $k = \pi$ (or, equivalently, around $x = \pi$). I'm working on it now, and will let you know very soon. – Costas Vlachos Oct 08 '20 at 10:23
  • Success! As Claude correctly pointed out, taking into account the symmetry around $\pi$ achieved a near-perfect match between the true inverse and the rational approximation, in the entire range of values for $k$, that is $0\leq k \leq 2\pi$. See these graphs, which clearly illustrate the quality of the match. @ClaudeLeibovici, thank you so much for your help and insight into the problem. Although my original questions still remain unanswered, your approach has provided a very useful and very accurate solution to the practical problem at hand. – Costas Vlachos Oct 08 '20 at 12:12
  • @DrCeeVee. What would be nice is to include in your post not the last graphs but the difference between the exact and approximate inverse. By the way, if this answer your question, may be you could accept it in order other users know that it is to some extent a valid solution. I have other ideas I am working on. – Claude Leibovici Oct 09 '20 at 06:32
  • I've computed the errors of the two approximate inverses, that is, for $k = x - \sin x$, and for the original function $V\left(\varphi\right)$, which can be seen in these graphs. As you can see, the errors are very small. Notice the lack of symmetry in the second (bottom) plot. This lack of symmetry also exists in my earlier function plots of $V\left(\varphi\right)$, and is due to the square root that is present in $V\left(\varphi\right)$. – Costas Vlachos Oct 09 '20 at 10:23
  • @DrCeeVee. What do you think about my third answer ? Again, add these plots to the post, please. – Claude Leibovici Oct 09 '20 at 10:26
1

Another solution

Built around $x=0$ the simplest Padé approximant is $$x-\sin(x) \sim \frac {x^3}{6+\frac{3 }{10}x^2}$$ for the zero of function $$f(x)=x-\sin(x)-k$$ we can generate the starting point $$t=\frac{k}{10} \left(1+2 \cosh \left(\frac{1}{3} \cosh ^{-1}\left(1+\frac{3000}{k^2}\right)\right)\right)$$

Using the first iteration of the original Householder method would give $$x\sim t+\frac{3 \left(\left(4 (k-t)^2+13\right) \sin (t)+16 (t-k) \cos (t)+16 k-16 t-8 \sin (2 t)+\sin (3 t)\right)}{72-8 (k-t) (\sin (2 t)-3 \sin (t))+\left(4 (k-t)^2-95\right) \cos (t)+24 \cos (2 t)-\cos (3 t)}$$ which seems to be much better than the previous ones (for seven significant figures the results are identical).

$$\left( \begin{array}{ccc} k & \text{estimate} & \text{solution} \\ 0.1 & 0.853750157 & 0.853750157 \\ 0.2 & 1.083691880 & 1.083691880 \\ 0.3 & 1.248515468 & 1.248515468 \\ 0.4 & 1.382284134 & 1.382284134 \\ 0.5 & 1.497300389 & 1.497300389 \\ 0.6 & 1.599585617 & 1.599585617 \\ 0.7 & 1.692592064 & 1.692592064 \\ 0.8 & 1.778505826 & 1.778505826 \\ 0.9 & 1.858809984 & 1.858809984 \\ 1.0 & 1.934563212 & 1.934563212 \\ 1.1 & 2.006551534 & 2.006551534 \\ 1.2 & 2.075377185 & 2.075377185 \\ 1.3 & 2.141513666 & 2.141513662 \\ 1.4 & 2.205341448 & 2.205341441 \\ 1.5 & 2.267172019 & 2.267172009 \\ 1.6 & 2.327264597 & 2.327264581 \\ 1.7 & 2.385838062 & 2.385838038 \\ 1.8 & 2.443079689 & 2.443079654 \\ 1.9 & 2.499151659 & 2.499151610 \\ 2.0 & 2.554196019 & 2.554195953 \\ 2.1 & 2.608338518 & 2.608338428 \\ 2.2 & 2.661691618 & 2.661691498 \\ 2.3 & 2.714356902 & 2.714356747 \\ 2.4 & 2.766427025 & 2.766426827 \\ 2.5 & 2.817987310 & 2.817987063 \\ 2.6 & 2.869117087 & 2.869116786 \\ 2.7 & 2.919890822 & 2.919890466 \\ 2.8 & 2.970379096 & 2.970378689 \\ 2.9 & 3.020649454 & 3.020649011 \\ 3.0 & 3.070767176 & 3.070766727 \\ 3.1 & 3.120795977 & 3.120795577 \end{array} \right)$$

A simpler could be given using instead the first iteration of the original Halley method $$x \sim t+\frac{4 \sin ^2\left(\frac{t}{2}\right) (k-t+\sin (t))}{(k-t) \sin (t)+(\cos (t)-4) \cos (t)+3}$$

  • I've just tested the Padé approximation approach. As expected, the results are excellent. The Padé/Householder method's absolute error is under $4.52\times10^{-7}$ almost everywhere, except the edges (around $k=0$ and $k=2\pi$, which are actually the same point), where there are some spikes, which I assume are due to numerical errors in evaluating the gigantic equation involved—I haven't optimised the order of operations in any way. The Padé/Halley method is about 2.5 orders of magnitude worse (absolute error is under $1.68\times10^{-4}$), but still very good. – Costas Vlachos Oct 11 '20 at 07:16
  • I'm including graphs of the above results, to better illustrate the magnitude and shape of the errors. I've plotted them in logarithmic scale in the $y$ axis, so that the differences are more clearly shown. For the practical problem at hand, I would use the rational approach, as is gives near-perfect results, while being simple to implement, very well-behaved at the edges, without exhibiting sensitivity in its numerical evaluation. I must say that I wasn't aware of the Padé approximation method, and was impressed by its excellent accuracy. Thanks Claude. – Costas Vlachos Oct 11 '20 at 07:33
  • @DrCeeVee.Again, put the graphs in the post. It would be a pitty to loose them. – Claude Leibovici Oct 11 '20 at 07:40
  • Yes, you're right, I will include all graphs in the original question. Just very busy... Will do it later on today. – Costas Vlachos Oct 11 '20 at 07:43
0

First of all, many thanks to all who replied with answers and comments. I've decided not to mark any of the provided answers as "accepted", since my questions were not really answered, but rather, approximations of the inverse of $V\left(\varphi\right)$ were provided. The answer by @PaulSinclair was, perhaps, the most relevant of all in attempting to answer my questions.

Shortly after posting here, I also posted the same questions to another relevant Stack Exchange community, MathOverflow, where I received the answers I was looking for, albeit not in the clarity I was hoping for (which may actually be impossible, due to the difficulty of the problem). Referring to the answers of my post at MathOverflow, what follows is a summary of the results I obtained.


Question — Does a closed-form expression for $\varphi\left(V\right)$ exist?

Answer — No, it does not exist (but see below).


Question — What is the formal reason for it? Is there a way to show/prove that such solution does not exist?

Answer — As shown in the comment by @user2661923, and by Pietro Majer at MathOverflow, the problem can be reduced to the inversion of function $k=x-\sin(x)$. Rob Corless at MathOverflow notes that this function exactly matches Kepler's Equation, $M=E-e \sin(E)$, where $e=1$. Rob Corless also adds that it is believed that Kepler's equation does not have any closed-form solution (let alone an elementary solution).

Piyush Grover at MathOverflow comments that "Kepler's equation is one of the most studied equations in the history of science. If there was a closed-form solution, it would have been found by now. There are papers about its approximations appearing to this day!"

Furthermore, the MathOverFlow accepted answer by Timothy Chow mentions that the question of whether the inverse of $k=x-\sin(x)$ has a closed-form solution may be answered by investigating if there can exist a solution that lies in a so-called Liouvillian extension of $\mathbb{C}(x)$, the field of rational functions of $x$ with complex coefficients. This includes any function that can be obtained via a finite number of applications of addition, subtraction, multiplication, division, taking $n$th roots, exponentiation, taking logarithms, and since we're working over the complex numbers, trigonometric functions and their inverses are included as well.

Timothy Chow expands further, providing a relevant theorem and several sources that support the above argument, and concludes by providing a source, in which it is mentioned that Liouville himself already knew that the solution to Kepler's equation (i.e., the solution to our problem) is not Liouvillian (i.e., there is no closed-form solution, as defined above).

Although I have not yet managed to fully decipher all the results provided, probably due to my lack of experience and knowledge in such advanced topics of mathematics—I'm not a mathematician, but an electronics engineer—I believe we now have sufficiently strong evidence to support the answer to my first question above, i.e., that there is no closed-form expression for $\varphi\left(V\right)$.


Approximations of the inverse of $k=x-\sin(x)$, and of $\varphi\left(V\right)$.

Among the answers I received here in Mathematics Stack Exchange, of particular interest and practical significance are those of @ClaudeLeibovici, who computed approximations of the inverse of $k=x-\sin(x)$ using a variety of methods, as listed below:

  1. Taylor expansion approximation (at various points in $[0,\pi]$),
  2. Rational approximation,
  3. Padé approximation (using the Householder and Halley methods).

You can refer to his specific answers for more details on each approach. I have evaluated all of his methods, and have found them to be of good to excellent accuracy, and extremely useful in practical applications, as they effectively provide closed-forms for very accurate approximations of $\varphi\left(V\right)$, enabling it to be evaluated in real-time by an embedded, microcontroller-based system, for example.

The figure below shows the errors of each approximation, for the reduced problem of inverting $k=x-\sin(x)$ (top plots), and for the complete solution to the practical problem, i.e., the computation of $\varphi\left(V\right)$ (bottom plots). Please refer to the comments I've made to each of Claude's answers, for some finer points regarding the implementation of each method.

enter image description here

  • see https://math.stackexchange.com/questions/1053472/how-to-solve-keplers-equation-m-e-varepsilon-sin-e-for-e/4732665#4732665 – IV_ Jul 08 '23 at 11:54